You are on page 1of 139

Electromagnetism

Course Instructor: Mesfin Tadesse (PhD)


Department of Physics, AAU
1
Content
 Coulomb’s Law
 Electric Fields
 Electric Potential
 Current, Resistance and Ohm’s Law
 Electric Power
 Equivalent Resistance and Kirchhoff’s Laws
 Magnetic Field and Magnetic Flux

2
Coulomb discovered an inverse
square law, like gravitation

The force acted along the line of


the charge centers

Its magnitude is

• proportional to the magnitudes of both


charges
• Inversely proportional to the square of
the distance between the charge centers
3
Experimentally, with r in meters (m), F in newtons
(N), and Q in Coulombs (C), it is found that
k = 9 x 109Nm2/C2
That is, two charges 1 coulomb each and 1 meter
apart, repel each other with a force of
9,000,000,000N!
Since the Coulomb is a very large unit, we usually
use the microcoulomb (μC):

1 μC = 10-6 C

4
Coulomb’s Law: Example

The simplest (Bohr) model of the hydrogen atom has an


electron circling a proton at a distance of about 0.5 x 10-10 m.

The electron charge has been determined experimentally to


be about -1.6 x 10-19 C.
Then, the electrostatic force holding the electron in orbit is:
( −1.6 10−19 )
2
2
e
F = k 2 = 9 109  = 9.2 10−8 N
r (.5 10 )
−10 2

The electron mass is about 10-30 kg, so its acceleration is about


1023 m/s2.

From ac = v2/r, v is about 2 x 106 m/s, around 1% of the speed of


light.
5
Check your understanding
Object A has a charge of +2μC and object B
has a charge of +6μC. Which statement is
TRUE about the electric forces on the objects:
A. 𝑭𝐴𝐵 = −3𝑭𝐵𝐴
𝑄
B. 𝑭𝐴𝐵 = −𝑭𝐵𝐴
C. 3𝑭𝐴𝐵 = −𝑭𝐵𝐴 A
3𝑄
D. 𝑭𝐴𝐵 = 3𝑭𝐵𝐴
E. 𝑭𝐴𝐵 = 𝑭𝐵𝐴 B
F. 3𝑭𝐴𝐵 = 𝑭𝐵𝐴

6
Superposition
 The total electric force on a charge 𝑄3 from two
charges 𝑄1 and 𝑄2 is the vector sum of the electric
forces from each charge as shown in the diagram.

 The principle of superposition is not valid for nuclear forces.


7
8
9
Visualizing The Electric Field
(vector diagram)

10
Visualizing The Electric Field
(lines of force)

11
Definition of The Electric Field
The strength 𝐸 of an electric field is
defined by bringing in a test charge 𝑞 and
measuring the electric force per unit
charge acting on the test charge.

• The test charge should be infinitely small so that


𝐹Ԧ its introduction in the electric field does not alter
𝐸= the configuration of the charges forming the field
𝑞 • E.g., charges on a conductor move if the test
charge is big enough to change the field to be
measured.

12
The Electric Field

Test charge q

13
Test charge q

 The field strength of Q does not


depend on the test charge q.

14
15
The Dipole Field

 The field lines near a


charge are dominated by
that charge
 The field lines cross at
right angles the line
perpendicular to the dipole
axis and midway between
the charges.

16
 Two equal positive charges Q
are placed on the y-axis at
equal distances d on opposite
sides of the origin.
 Show that the electric field on
the x-axis has magnitude

2𝑘𝑄𝑥
𝐸= 3
𝑥 2 + 𝑑2 2

 Where on the x-axis is the


field maximum?
17
Field Lines of Two Equal Charges
 The field is zero midway between the charges.

18
Solution

+Q Q’=?
x
-a 0 a 2a 3a
Example:
Two point charges are located
on the x axis. The first is a charge
+Q at x = -a. The second is an
unknown charge located at x =
+3a. The net electric field these
charges produce at the origin
has a magnitude of 2kQ/a2.
What are the two possible
values of the unknown charge?

19
Electric Potential Energy
Imagine a charge q0 placed in a uniform electric field: F = 𝑞0 E

When the charge is released, the field moves it from


A to B, doing work:

W = 𝐹𝑑 = 𝑞0 𝐸𝑑

Let UA = electric PE of the charge at point A


and UB = electric PE of the charge at point B
Then, by definition:

W = q 0 Ed = U A − U B = −U
If we define 𝑈𝐵 = 0, then 𝑈𝐴 = 𝑞0𝐸𝑑 is the electric
potential energy the charge has at point A.
Electric Potential: Unit
The quantity 𝐸𝑑 in 𝑊 = 𝑞𝐸𝑑 is called electric potential, V.

Thus, 𝑊 = 𝑞𝑉 or 𝑊 = −𝑞Δ𝑉 (Δ𝑉 is potential difference.)

A unit of energy that is commonly used in atomic and


nuclear physics is the electron-volt (eV)

One electron-volt is defined as the


energy a charge-field system gains or
loses when a charge of magnitude e 1 eV = 1.60 x 10-19 J
(electron or proton) is moved through
a potential difference of 1 volt.
Potential Energy of Two Point Charges

1 𝑄𝑞0
𝑈 𝑟 =
4𝜋𝜖0 𝑟

NB: The point at infinity is chosen as a zero reference


point: U() = 0
Potential Energy: Example
 Two balls of mass m=1 kg each and carrying charge Q=1 C each are
fixed at a distance r=1 m from each other. Find the final velocity of
the balls after they have been let go.

SOLUTION
Apply conservation of total energy
𝐸𝑏𝑒𝑓𝑜𝑟𝑒 𝑟𝑒𝑙𝑒𝑎𝑠𝑒 = 𝐸𝑎𝑓𝑡𝑒𝑟 𝑟𝑒𝑙𝑒𝑎𝑠𝑒
𝐾1𝑖 + 𝐾2𝑖 + 𝑈𝑠𝑦𝑠,𝑖 = 𝐾1𝑓 + 𝐾2𝑓 + 𝑈𝑠𝑦𝑠,𝑓
𝑄2 1 1
0+0+𝑘 = 𝑚𝑣 + 𝑚𝑣 2 + 0
2
𝑟 2 2
𝑘 𝑄2
𝑣= ≈ 100 𝑘𝑚/𝑠
𝑚 𝑟
NB: the escape velocity for Earth is about 11 km/s
23
Potential Energy: Example
Calculate the electric potential energy of two protons separated
by a typical proton-proton intranuclear distance of 2x10-15 m.
𝑒2 1.6 × 10−19 2
9
𝑈 = 𝑘 = 9 × 10 × = +1.15 × 10−13 𝐽
𝑟 2 × 10−15

What is the meaning of the + sign in the result? ( +1.15x10-13 J)

Calculate the electric potential energy of a hydrogen atom


(electron-proton distance is 5.29 x10-11 m).
𝑒2 − 1.6 × 10−19 2
𝑈 = 𝑘 = 9 × 109 × = −4.36 × 10−18 J
𝑟 5.29 × 10−11

What is the meaning of the - sign in the result? Is that a small energy?
Potential Energy: Example
The potential energy of the hydrogen atom is about -4.36x10-18
joules. How many electron volts is that?
 1 eV 
U = -4.36 10 J = ( -4.36 10 J ) 
-18 -18
-19 
 -27.2 eV
 1.6 10 J 
What is the kinetic energy of the electron given that the ground-
state energy of the hydrogen atom is -13.6 eV?
2 2
e ke
K = 12 mv 2 = 12 mac r = 12 Fc r = 12 k 2 r =
r 2r

K=
9
(
9 10  1.6 10 )
−19 2

1eV
= 13.6eV
2  0.53 10 −10 1.6 10 −19

Add KE+PE to get ground state energy.


Potential Energy of Multiple Charges
 If there are more than two
charges, first find U for each
pair of charges and then add
them

 For three charges:

𝑞1 𝑞2 𝑞1 𝑞3 𝑞2 𝑞3
𝑈=𝑘 + +
𝑟12 𝑟13 𝑟23

 The result is independent of


the order of the charges
Example: Find the total potential energy of the system of
three charges.
y

q3
P q1 q 2 q1 q 3 q 2 q 3
U = k + +
3m r12 r13 r23

q1 4m q2 x

9
1 × 10−6 −4 × 10−6 1 × 10−6 3 × 10−6 −4 × 10−6 3 × 10−6
U = 9 × 10 + +
4 3 5

U = −2.16 × 10−2 J
Electric Potential Difference
 Suppose a unit positive charge is
moved between two points A and
B in an electric field.
1 B
 The electrical PD is then the work
2 done to move the unit charge
between the two points
A
𝑊𝐴→𝐵 𝑈𝐵 − 𝑈𝐴
𝑉𝐵 − 𝑉𝐴 = =
𝑞0 𝑞0
NB:
 The potential V is independent of the amount of charge being moved
 The PD (V) is path independent
 The unit for electric potential is volt (V): 1 V = 1 J/C
The Electric Potential of a Point Charge
 For the point charge:

1 𝑄
𝐸(𝑟)
Ԧ = 2
𝑟Ƹ
4𝜋𝜀0 𝑟

 Then

1 𝑄
𝑉(𝑟) =
4𝜋𝜀0 𝑟
The dotted circles
(spheres in 3D) show
equipotential regions
Electric Potential with
Multiple Charges
 The electric potential due to several point charges is the
sum of the potentials due to each individual charge
 This is another example of the superposition principle
 The sum is algebraic:
V=?
1 qi
V= 
4 0 i ri

 V = 0 at r = ∞ ri
qi
Example: A 1 C point charge is located at the origin and a -4 C
point charge 4 meters along the +x axis. Calculate the electric
potential at a point P, 3 meters along the +y axis.

y
qi  q1 q 2 
VP = k  = k  + 
P i ri  r1 r2 
3m 9  1×10 -6
-4×10 -6

= 9×10  + 
x  3 5 
q1 4m q2
= - 4.2×103 V
E and V of
a conducting sphere
 All charges reside on the surface of the
metal
 The electric potential is
 Constant inside the sphere
 a function of r out side the sphere
 The electric field is
 Zero inside the metal
 a function of r2 outside
 The effect of a charge on the space
surrounding it:
 The charge sets up a vector electric field
which is related to the force
 The charge sets up a scalar potential
which is related to the energy
Electric Current, the definition
 Assume charges are moving
+ +
perpendicular to a surface of
area A
A
+ +

+ +
 If ΔQ is the amount of charge
that passes through A in time Δt,
then the current is I

 SI unit of current is ampere (A)


Q  1A=1C/s
I avg =  Conventional current is in the
t same direction as the flow of
positive charges
Motion of charge carriers in a conductor
 The actual charge carrier is the electron
 The zigzag lines represents the motion
of a charge carrier in a conductor
 The net drift speed is small
 The sharp changes in direction are due
to collisions

 The net motion of electrons is opposite the direction of the


electric field
 Conventional current is in the direction of the electric field.
 The drift speed for a 12-gauge copper wire carrying a current of
10.0 A is 2.23 x 10-4 m/s (Assignment)
Conductivity, the definition
 For some materials, the current density is directly
proportional to the field, that is
 
J = E  Empirical law
 Valid for Ohmic Materials

 This relation is called Ohm’s Law


 The constant of proportionality, σ, is called the
conductivity of the conductor
 Another often used parameter is resistivity, ρ, and is
just the inverse of the conductivity: ρ = 1/σ
Resistance and a more popular form
of Ohm’s Law 𝑙
From Ohm’s Law: 𝐽 = 𝜎𝐸 E
1
One has: 𝐸 = 𝐽 = 𝜌𝐽
𝜎
For a section of wire of length 𝑙 and cross section A:
𝐼 𝑙
Δ𝑉 = 𝐸𝑙 = 𝜌𝐽𝑙 = 𝜌 𝑙 = 𝜌 𝐼
𝐴 𝐴
Now we define a new parameter, the resistance of this
section of the wire, R:
𝑙 Ohm’s Law Δ𝑉
𝑅≡𝜌 becomes
Δ𝑉 = 𝑅𝐼 or 𝑅 ≡
𝐴 𝐼
Ohmic Material, Graph
 For an ohmic device
 The resistance is constant
over a wide range of voltages

 The relationship between


current and voltage is linear

 The slope is related to the


resistance
Nonohmic Material, Graph
 Nonohmic materials
are those whose
resistance changes with
voltage or current

 The current-voltage
relationship is
nonlinear
Resistivity Values and the temperature coefficient α
Resistors
A device made to have certain
resistance value is called a resistor.

(Almost) all materials


have resistance
3.8x1021 electrons pass through a point in a wire
in 4 minutes. What was the average current?

Solution:
Q Ne
Iav = =
t t

Iav =
( 3.8  10 21
)(1.6  10 −19
)
( 4  60 )

Iav = 2.53A
A flashlight bulb carries a current of 0.1 A. Find the
charge that passes through the bulb in 0.5 Seconds.

Solution:

I = ΔQ/ΔT => ΔQ = I × ΔT = 0.1C/s × 0.5s = 0.05 C

How many electrons does this correspond to?

ΔQ = Ne => N = ΔQ/e = 0.05C / (1.6×10-19 C)


= 3.1×1017 electrons
Rank the currents from lowest to highest and show the
direction of conventional current:

Figure a b c d
Rank 1 2 2 3
Direction of conventional Right Left Right Left
current
Consider a simple V-R circuit comprising a light bulb. Assume
there is a 1.5-volt battery and the light bulb draws a current of
0.2 Amps. Find the R of the light bulb filament.

Solution: R = ΔV/I = 1.5V/0.2 A = 7.5 Ω

Calculate the current density in a gold wire at 20°C if an electric


field of 0.740 V/m exists in a wire.

Solution:

𝐸 0.740𝑉Τ𝑚
𝐽 = 𝜎𝐸 = = = 3.03 × 107 𝐴/𝑚2
𝜌 2.44×10−8 Ω𝑚
Suppose you want to connect your stereo to remote speakers.
(a) If each wire must be 20 m long, what diameter copper wire
should you use to make the resistance 0.10  per wire.
(b) If the current to each speaker is 4.0 A, what is the voltage
drop across each wire?

Solution: (a) Solution: (b)

R = L / A V=IR
A = L / R V = (4.0) (0.10)
 (d/2)2 = L / R V = 0.4 V
d = 2 ( L / R )½
d = 2 [ (1.68x10-8) (20) /  (0.1) ]½
d = 0.0021 m = 2.1 mm
The 12-gauge copper wire in a home has a cross-sectional area of
3.31 x 10-6 m2 and carries a current of 10 A. Calculate the
magnitude of the electric field in the wire.

Solution: I
E = J = 
A
(1.72 10−8   m) (10 C/s )
E=
(3.3110−6 m 2 )
E = 5.20 10−2 V/m
Question: What is the source of this electric field?

Question: Are we still assuming the electrostatic case?

Here, in this particular example, it is safe to use V=Ed.


Resistor Connections
In series
𝐼 = 𝐼1 = 𝐼2 V1 V2
Condition: Δ𝑉 = Δ𝑉1 + Δ𝑉2

Δ𝑉 Δ𝑉1 + Δ𝑉2 Δ𝑉1 Δ𝑉2


𝑅𝑒𝑞 ≡ = = + ≡ 𝑅1 + 𝑅2
𝐼 𝐼 𝐼1 𝐼2

In parallel
Condition: 𝐼 = 𝐼1 + 𝐼2 V1

Δ𝑉 = Δ𝑉1 = Δ𝑉2
1 𝐼 𝐼1 + 𝐼2 𝐼1 𝐼2 1 1 V2
≡ = = + ≡ +
𝑅𝑒𝑞 Δ𝑉 Δ𝑉 Δ𝑉1 Δ𝑉2 𝑅1 𝑅2
Example
 The 8.0- and 4.0- resistors
are in series and can be replaced
with their equivalent, 12.0 

 The 6.0- and 3.0- resistors are


in parallel and can be replaced
with their equivalent, 2.0 

 These equivalent resistances are


in series and can be replaced
with their equivalent resistance,
14.0 
Calculate the equivalent resistance of the resistor
“ladder” shown. All resistors have the same resistance R.

R R R
A

R R R

B
R R R
Series

A
R
R 3R
R
B
Parallel

3
R 3R 4 R

B
Series

A
R
3 11
4 R 4 R
R
B
Parallel

11
R 4 R 11
15 R

B
Series

A
R
11 41
15 R 15 R
R
B
All done!

A Exercise
After the lessons on Kirchhoff’s
41 rules, calculate the equivalent
15 R resistance between A and B
using Kirchhoff’s two rules: the
junction rule and the loop rule
B
For the circuit below, calculate the current drawn from
the battery and the current in the 6  resistor.

10 

R?
8
6
I?
3 9V
8
1
9V
Solution
10 

8

6
3
8

1 9V

Replace the parallel combination (green) by its


equivalent.
Remember that everything inside the green box is equivalent to a single
resistor.
10 

4

6
3

1 9V

Replace the series combination (blue box) by its


equivalent.
10 

10 
3

1 9V

Replace the parallel combination (orange) by its


equivalent.
Now calculate the current drawn from the battery.

5

3

1 9V

 9
I= = =1A
R1 + R2 + R3 9
Calculate the current through the 6-Ω resistor.
10 

A B

I1
6 4
3

I=1A

1 9V
Start at B and follow the blue Then follow the green path
path to point A
VA − 6 I1 − 4 I1 = VB  VAB = −10 I1
VB − 3 1 + 9 − 11 = VA VAB − VBA 5
I1 = = = = 0.5 A
VBA = VA − VB = 5V − 10 − 10 10
Rule 1: Kirchhoff’s Junction Rule
 Junction Rule, from charge
conservation:
 The sum of the currents at any
junction must equal zero
 Mathematically:

 I =0
junction Junction
 The example on the left figure:
 I1 - I2 - I3 = 0
Rule 2: Kirchhoff’s Loop Rule
 Loop Rule, from energy
conservation:
V1 V2
 Choose your loop
 The sum of the potential differences
Loop direction
across all elements around any
closed circuit loop must be zero
 Mathematically:

 V = 0
closed
Remember two things:
1. A battery supplies power. Potential
loop rises from the “–” terminal to “+”
terminal.
 Pay attention to the sign (+ or -) of
the potential changes when you go 2. Resistors dissipate power.
around a chosen loop direction. Potential falls in the direction of
the current through the resistor.
Example
Step 1: Choose and mark the loops.
Step 2: Choose and mark current directions. Mark the potential change on
resistors.
Step 3: Apply junction rule:
– +
I1 + I2 − I3 = 0 I2
L1
Step 4: Apply loop rule: I1 I3
+ –
L1: +2.00I3 − 12.0 + 4.00I2 = 0
L2
L2: − 8.00 − 2.00I3 − 6.00I1 = 0
– +
Step 5: Solve the three equations for
the three variables.
18 28 10
𝐼1 = − 𝐴, 𝐼2 = 𝐴, 𝐼3 = 𝐴
11 11 11
Calculate I, Vab, and Vba.

Use Kirchhoff's loop rule  1 −  2 − IR1 − IR2 = 0


3
9 − 6 − 5I − 10 I = 0 I = = 0.2 A
15
 − IR1 − IR2 = 0 Va + 9 − 5(0.6) = Vb
Va + 9 − 6 − 5(0.2) = Vb
9 − 5I − 10 I = 0 Vab = Vb − Va = 6V
Vab = Vb − Va = 2V
9
I = = 0.6 A
15 Vb − 10(0.6) = Va
Vb − 10(0.2) = Va
Vba = Va − Vb = −6V
Vba = Va − Vb = −2V
Example h
30 
I1
40  I3 1 2 = 45 V
Find aa dd
b c
I1, I2, and I3.
20 
I2
1 = 85 V 1
g f e

Junction a: I3 – I1 – I2 = 0 - - eq. 1 Junction d gave no


new information, so
Junction d: -I3 + I1 + I2 = 0 we still need two
more equations.
30 
h
I1
40  I3 1 2 = 45 V
a d
b c

20 
I2
1 = 85 V 1
g f e

There are three loops. Loop 1 Loop 2 Loop 3

Any two loops will produce independent equations. Using the


third loop will provide no new information.
Reminders:
I +-
V is - V is +
loop loop

The “green” loop (a-h-d-c-b-a):


(- 30 I1) + (+45) + (-1 I3) + (- 40 I3) = 0
- 30 I1 + 45 - 41 I3 = 0 --eq. 2

The “blue” loop (a-b-c-d-e-f-g):


(+ 40 I3) + ( +1 I3) + (-45) + (+20 I2) + (+1 I2) + (-85) = 0
41 I3 -130 + 21 I2 = 0 --eq. 3
Three equations, three unknowns; the rest is “algebra.”
Make sure to use voltages in V and resistances in . Then currents will be in A.
I3 – I1 – I2 = 0
Collect the three equations: - 30 I1 + 45 - 41 I3 = 0
41 I3 -130 + 21 I2 = 0

– I1 – I2 + I3 = 0
Rearrange to get variables in
“right” order: - 30 I1 - 41 I3 + 45 = 0
21 I2 + 41 I3 -130 = 0

There are many valid sets of steps to solving a system


of linear equations. Any that works is acceptable.

Don’t forget to show units in the final


answer
Finding the Currents
– I1 – I2 + I3 = 0 Eq. 1
- 30 I1 - 41 I3 + 45 = 0 Eq. 2 From Eq. 1: I3 = I1 + I2
21 I2 + 41 I3 -130 = 0 Eq. 3

Substituting in Equations (2) and (3):


- 30 I1 + 0 I2 - 41 (I1 + I2 ) = - 45 - 71 I1 - 41 I2 = - 45 Eq. 4
0 I1 + 21 I2 + 41 (I1 + I2 ) = 130 41 I1 + 62 I2 = 130 Eq. 5

Solving Equations (4) and (5) simultaneously:

I1 = - 0.933 A Check the results:


I2 = +2.71 A Are these currents
correct? How could
𝐼3 = 𝐼1 + 𝐼2 = −0.933𝐴 + 2.71𝐴 = 1.78𝐴 you tell?
Model of a battery
 Two parameters, electromotive force
(emf), , and the internal resistance r, V
are used to model a battery.
 When a battery is connected in a
circuit, the electric potential measured
at its + and – terminals are called the
terminal voltage V, with V =  – Ir
 If the internal resistance is zero, the
terminal voltage equals the emf . battery
 The internal resistance, r, does not V
change with external load resistance R.

load
Battery power
Power generated by the battery battery
(through chemical reactions): V

p = ε  I = (R + r )  I 2
load
The power the battery delivers to the load (efficiency):
pload R
pload = V  I = R  I 2
efficiency = =
p R+r
The maximum power the battery can deliver to a load
R
ε = (R + r )  I We have pload = ε 2
From pload = R  I
2
and
(R + r )2
Where the emf 𝜀 is a constant once the battery is given.

dpload  1 2R  2 R = r is the condition for maximum pload,


From = − 3 
ε =0 or power delivered to the load.
 (R + r ) (R + r ) 
2
dR
Battery power
One can also obtain this result
from the plot of battery
V
𝑅 2
𝑃𝑙𝑜𝑎𝑑 = 𝜀
(𝑅 + 𝑟)2
load
When 𝑅 = 𝑟, 𝑃𝑙𝑜𝑎𝑑 becomes maximum.

The efficiency of the battery at this


point is 50%.

𝑝𝑙𝑜𝑎𝑑 𝑅
efficiency = =
𝑝 𝑅+𝑟
Example
 For maximum power delivered to the external circuit
by the battery, the internal resistance of battery r is
A. 10R
B. 4R/9
C. R/8
D. 10R/9

76
Magnets and the Magnetic Field
 Magnets always have two poles: N and S
poles.
 Like poles repel, unlike poles attract.
 Outside a magnet, field lines start from
the north pole, end at the south pole.
 The magnetic field is a vector, and is
symbolized by 𝐵

• Field lines can be traced out by a


small compass.
• SI unit of magnetic field: tesla (T)
• Non-SI unit: gauss (G)
• 1 T = 104 G
Magnetic field
Field lines of a of N and S poles of N and N poles
single bar magnet

Electric field
What generates the magnetic field?
 The magnetic field is generated by:
 moving charges
 currents
 changing electric field

⚫ The field of a permanent


magnet (ex. the bar magnet) is
generated by currents in the
atoms of the material.
Magnetic force on moving charge
The formula: Compare with
 
𝐅Ԧ𝐵 = 𝑞𝐯 × 𝐁 FE = q E

The direction of the force is determined


by the charge and the vector product of
the velocity and the magnetic field.

The magnitude: For a positive charge


𝐹𝐵 = 𝑞𝑣𝐵 sin 𝜃

𝐹𝐵 = 𝑞𝑣𝐵 When velocity and field are perpendicular.


Which way is the magnetic force?
In a lightning strike, there is a rapid
movement of negative charge from
the cloud to the ground. In what
direction is a lightning strike
deflected by Earth’s magnetic
field?

Answer: toward the west


Example
 Calculate the force on a positive charge of 20-nC due to the Earth’s
magnetic field (5 × 10–5 𝑇), if it moves at 10 m/s due west in a
place where the Earth’s field is due north parallel to the ground.

 Solution
Angle between 𝒗 and 𝑩 is 90° so
sin θ = 1.

𝐹 = 𝑞𝑣𝐵 sin 𝜃
𝐹 = (20 × 10–9 𝐶)(10 𝑚/𝑠)(5 × 10–5 𝑇)
𝐹 = 1 × 10–11 𝑁
82
Force on a Charged Particle
 The magnetic force causes a
centripetal acceleration,
changing the direction of the
velocity of the particle
 Equating the magnetic and
centripetal forces:
𝑚𝑣 2
𝐹𝐵 = 𝑞𝑣𝐵 =
𝑟
 Solving for r:
𝑚𝑣
𝑟=
𝑞𝐵

𝑟 is proportional to the linear momentum of the particle and


inversely proportional to the magnetic field
Example
A proton with velocity v=1×106 m/s is in a uniform B-
field of 0.2 T. Find r:

Solution
r = mv/qB
r = (1.67×10-27 x 1×106) / (1.6×10-19 x 0.2)
r = 0.052 m = 5.2 cm

What if the velocity is parallel to B?


Motion of a Particle, General
 If a charged particle
moves in a magnetic
field at some arbitrary
angle with respect to the
field, its path is a helix
 Same equations apply,
with

𝑣⊥ = 𝑣𝑦2 + 𝑣𝑧2
Magnetic Force on a Current Carrying
Conductor, a wire
 A force is exerted on a
current-carrying wire
placed in a magnetic field

 The direction of the force


is given by the right-hand
rule
Force on a Wire, the formula
Magnetic force on each moving
charge in the wire

𝐹 = 𝑞𝑣𝑑 𝐵 sin 𝜃

The total force on the wire

𝐿
𝐹 = 𝑁𝑞𝑣𝑑 𝐵 sin 𝜃 = Δ𝑄 𝐵 sin 𝜃
Δt
𝐹 = 𝐼𝐿𝐵 sin 𝜃 𝐅Ԧ𝐵 = 𝐼 𝐋
Ԧ ×𝐁
Torque on a Current Loop
 The rectangular loop carries a
current I in a uniform magnetic field F
 No magnetic force acts on sides 1 & 3.
 The magnitude of the magnetic
force on sides 2 & 4 is F
 F 2 = F4 = I a B
 The direction of F2 is out of the page;
The direction of F4 is into the page
 The forces form a couple. Front view
 The couple produces a torque about
a vertical axis through the center of
the loop
 Or around point O (bottom view)
 The loop rotates about an axis
through point O.
bottom view
Torque on a Current Loop, Equation
 The maximum torque is found by:

𝑏 𝑏 𝑏 𝑏
𝜏𝑚𝑎𝑥 = 𝐹2 + 𝐹4 = 𝐼𝑎𝐵 + 𝐼𝑎𝐵 = 𝐼 𝑎𝑏 𝐵
2 2 2 2

 The area enclosed by the loop is 𝒜 = 𝑎𝑏, so

𝜏𝑚𝑎𝑥 = 𝐼𝒜𝐵

 This maximum value occurs only when the field is


parallel to the plane of the loop
Torque on a Current Loop, General

 Assume the magnetic field


makes an angle of q < 90o with a
line perpendicular to the plane
of the loop

 The net torque about point O


will be 𝜏 = 𝐼𝒜𝐵 sin 𝑞

 When the direction of the loop


area is defined, the torque can be
expressed in its vector format:

τ = 𝐼 𝒜Ԧ × B
Direction of a current loop and the Magnetic
Dipole Moment
 The right-hand rule can be used to
determine the direction of 𝒜Ԧ
 Curl your fingers in the direction of the
current in the loop
 Your thumb points in the direction of 𝒜Ԧ
 The product 𝐼 𝒜Ԧ is defined as the magnetic
(dipole) moment, 𝜇, Ԧ of the loop
 SI units: A · m2
 Torque in terms of magnetic moment:

𝜏Ԧ = 𝜇Ԧ × 𝐁
Example
 A proton is moving with a velocity 𝑣Ԧ = 𝑣0 𝑗Ƹ in a region of
uniform magnetic field. The resulting force is 𝐹Ԧ = 𝐹0 𝑖Ƹ If the
proton is initially shot perpendicular to the magnetic field, find
the magnitude and direction of the magnetic field?

 Solution
𝑣Ԧ = 𝑣0 𝑗Ƹ
𝐹
𝐹 = 𝑞𝑣𝐵 ⇒ 𝐵 =
𝑞𝑣
𝐹0
𝐵=
𝑒𝑣0
𝐵 𝐹Ԧ = 𝐹0 𝑖Ƹ

Ԧ can the
 For the same 𝑣Ԧ and 𝐹,
magnetic field have an x 𝐹0
𝐵= 𝑘෠ + 𝐵𝑦 𝑗Ƹ
component? A y component? 𝑒𝑣0
Example
 Find the maximum torque on a 100-turn square loop of
a wire of 10.0 cm on a side that carries 15.0 A of current
in a 2.00-T field.

Solution
The maximum torque corresponds to sin θ = 1, so
𝜏𝑚𝑎𝑥 = 𝑁𝐼𝒜𝐵
𝜏𝑚𝑎𝑥 = (100)(15.0 𝐴)(0.100 𝑚2 )(2.00 𝑇) = 30.0 𝑁𝑚.

93
Example
 A proton is moving at 1 × 104 m/s from left to right in a
magnetic field of 0.4 T that’s points up in the plane of the
page. Find the magnitude of the force vector. Find the
direction. What would the force be if the particle was an 𝑒 – ?

 Solution
 𝐹 = 𝑞𝑣𝐵 sin 𝜃 = 1.6 × 10−19 × 104 × 0.4 × sin 90°
 𝐹 = 6.4 × 10−16 𝑁

 Direction of force: out of the page towards you.

 Force–: Magnitude of force is same, but in the opposite direction.

94
Example
 An electron travels at 2 × 107 m/s in a plane perpendicular
to a 0.01 T magnetic field. Describe its path and find the
cyclotron frequency.

 Solution
The path is a circle of radius:
mv 9.1110 −31  2 107
r= = −19
= 1.14 cm
eB 1.602 10  0.01

 The cyclotron frequency is


eB 1.602 10 −19  0.01
f= = −31
= 2.8  10 8
cycles/s
2m 2  9.1110
A wire carrying current I consists of a semicircle of radius R and
two horizontal straight portions each of length L. It is in a region
of constant magnetic field as shown. What is the net magnetic
force on the wire?

y
       
B
        x
R
I        

L       L 

       

There is no magnetic force on the portions of the wire outside the magnetic field region.
First look at the two straight sections.

𝐹 = 𝐼𝐿𝐵 sin 𝜃

L ⊥ B, so
       
F1 = F2 = ILB F1 F2 B
       
R
Total force on the I        
straight parts is  L       L
y
       

Fst = F1 + F2 = 2ILB x
Fy
Total force:        
F1 F2 B
       
F = F1 + F2 + Fy R
I        

 L       L 
F = ILB + ILB + 2IRB y
       

F = 2IB (L + R ) x

This is as if we have a straight wire of length 2(L+R) immersed in


a magnetic field.

How would the result differ if the magnetic field were directed
along the +x direction?
A semicircular closed loop of radius R carries current I. It is in a
region of constant magnetic field as shown. What is the net
magnetic force on the loop of wire?

FC
y
       
B
        x
R
       

   I    

       

We calculated the force on the semicircular part in the previous example


(current is flowing in the same direction there as before).
FC = 2 I R B
Next look at the straight section. FC

       
𝐹 = 𝐼𝐿𝐵 sin 𝜃 B
       
R
       
L ⊥ B, and L=2R so
   I    
FS = 2IRB y
       
FS
x

Fs is directed in the –y direction (right hand rule).


Fnet = FS +Fc = -2IRB ˆj+2IRB ˆj = 0

The net force on the closed loop is zero!


This is true in general for closed loops in a uniform magnetic field.
Magnetic field of a Long, Straight Conductor
with current I
μo I
B=
2πa
 The magnitude of the field is constant on
any circle of radius a
 The direction of the field is determined by
the right-hand rule
 The magnetic field lines are circles
concentric with the wire
 The field lines lie in planes perpendicular
to wire
Example
Find the current in a long straight wire that would produce a
magnetic field twice the strength of the Earth’s field at a distance
of 5.0 cm from the wire. 𝐵𝐸 ≅ 5.0 × 10−5 𝑇.

Solution
𝜇0 𝐼
Solving 𝐵 = for the current I, we get
2𝜋𝑟

2𝜋𝑟𝐵 2𝜋𝑟
𝐼= = (2𝐵𝐸 )
𝜇0 𝜇0

2𝜋 × 5.0 × 10−2 𝑚 −5
𝐼= −7
2 × 5.0 × 10 𝑇 = 25 𝐴
4𝜋 × 10 𝑇𝑚/𝐴
102
Bio-Savart Law
 The B-field due to an element of current in a wire is
given by the Bio-Savart law:

𝜇0 𝐼𝑑 𝑙Ԧ × 𝑟Ƹ
𝑑𝐵 =
4𝜋 𝑟 2 I

dl θ
The B-field of a
straight line 𝒓ො r
follows from this x B
law by integration.
103
Magnetic field at the center of a circular loop of
wire with current I and radius R y
⚫ By Bio-Savart law:

𝜇0 Id𝑙Ԧ × 𝑟Ƹ 𝜇0 𝐼𝑑𝑙 x
𝑑𝐵 = 2
= 2
𝑘෠ 𝑟Ԧ
4𝜋 𝑟 4𝜋 𝑅

𝜇0 𝐼 𝜇0 𝐼
𝐵 = ෍ 𝑑𝐵 = ෠
𝑘 ෍ 𝑑𝑙 = 𝑘෠
4𝜋 𝑅 2 2𝑅

⚫ The magnitude of the magnetic field is

𝝁𝟎 𝑰
𝑩=
𝟐 𝑹
Example
A long, straight wire carries current I. A right-angle bend is made
in the middle of the wire. The bend forms an arc of a circle of
radius r. Determine the magnetic filed at the center of the arc.
Use the results already obtained:

1 𝜇𝑜 I 𝜇𝑜 I
Straight section: 𝐵 = =
2 2𝜋𝑟 4𝜋𝑟

1 𝜇𝑜 I 𝜇0 𝐼
Arc: 𝐵= =
4 2𝑟 8𝑟

μo I μo I μo I μo I  2 1 
The final answer: magnitude B= + + =  + 
4πr 8r 4πr 4r  π 2 
direction pointing into the page.
Magnetic Field of a Solenoid
 A solenoid is a long wire wound in the 𝑵
form of a helix. 𝑩 = 𝝁0 𝑰 = 𝝁0 𝒏𝑰
𝒍
 Each loop produces a magnetic field that
adds together to form the total field.

 A reasonably uniform magnetic field can


be produced in the space surrounded by
the turns of the wire

 The field lines in the interior are

 nearly parallel to each other

 uniformly distributed

 Outside the solenoid, the field is weak as


field lines from neighboring wires tend to
cancel.
What is the field inside a 2.00-m-long solenoid that has 2000 turns
and carries a 1600-A current?
Solution
Substituting known values into Equation 1.15, we get
𝑁 −7
𝑇𝑚 2000 𝑡𝑢𝑟𝑛𝑠
𝐵 = 𝜇0 𝐼 = 4𝜋 × 10 (1600 𝐴)
𝑙 𝐴 2.00 𝑚
= 2.01 𝑇.

107
Exercise:
A thin 10-cm long solenoid has a total of 400 turns of wire and
carries a current of 2 A. Calculate the magnetic field inside
near the center.

N
B = μ0 I

 -7 T  m  ( 400 )
B =  4 π ×10  (2 A )
 A  ( 0.1 m )

B = 0.01 T
Magnetic Force Between Two Parallel Wires
 The wires each carry steady currents
 The field 𝐁2 due to the current in wire
2 exerts a force on wire 1:
F1 = I1 ℓ B2
 Substituting the equation for 𝐁2 gives
𝜇𝑜 I1 I2
𝐹1 = ℓ
2𝜋a

 Check with right-hand rule:


 Currents in the same direction attract Force per unit length
each other 𝐹𝐵 𝜇𝑜 I1 I2
 Currents in opposite directions repel =
ℓ 2𝜋a
each other
This formula defines the
current unit , the Ampere.
Magnetic Flux Through a Plane
 Consider a plane of area A, its
ෝ makes an angle q with 𝐁.
direction 𝒏

The magnetic flux is


FB = BA cos q

 When the field is perpendicular to 𝐴Ԧ


F = 0 (figure a)

Ԧ
 When the field is parallel to 𝐴,
F = BA (figure b)
Magnetic Flux Through Any surface

 Magnetic flux through an element of


area Δ𝐴Ԧ is:
Δ𝐴Ԧ
θ
B
ΔΦ𝐵 = 𝐵⊥ ΔA = B cos 𝜃 Δ𝐴

SI unit: weber (Wb) = T.m2


B θ
Δ𝐴Ԧ
 The magnetic flux through a surface
bounded by a loop is found by adding
contributions from tiny squares.
𝑁

Φ𝐵 = ෍ 𝐵𝑖 𝑐𝑜𝑠 𝜃𝑖 𝛥𝐴𝑖
𝑖=1
Magnetic Flux: Examples
 What is the magnetic flux through
the loop of area A = 10 cm × 5 cm
shown in the figure if the magnetic
field has a strength of 0.5 T and
makes an angle of 30o with the
normal to the loop?

 Solution
Φ𝐵 = 𝐵𝐴 cos 𝜃
𝜱𝑩 = 𝑩⊥ 𝑨 = 𝑩𝑨 𝒄𝒐𝒔 𝜽 Φ𝐵 = 0.5 × 0.005 × cos 30°
Φ𝐵 = 2.2 × 10−3 𝑊𝑏

112
Example
A conducting circular loop of radius 0.250 m is placed in the xy-
plane in a uniform magnetic field of 0.360 T that points in the
positive z-direction, the same direction as the normal to the plane.
1. Calculate the magnetic flux through the loop.

Solution z
1. Φ𝐵 = 𝐵𝐴 cos 𝜃 = 𝐵 𝜋𝑟 2 cos 0°
Φ𝐵 = 0.360 𝑇 𝜋 × 0.2502 𝑚2 y
Φ𝐵 = 0.0706 Wb

113
Example
A conducting circular loop of radius 0.250 m is placed in the xy-
plane in a uniform magnetic field of 0.360 T that points in the
positive z-direction, the same direction as the normal to the plane.
1. Calculate the magnetic flux through the loop.
2. Suppose the loop is rotated clockwise around the x-axis, so the
normal direction now points at a 45.0° angle with respect to the
z-axis. Recalculate the magnetic flux through the loop.

Solution z
2. Φ′𝐵 = 𝐵𝐴 cos 𝜃 = 𝐵 𝜋𝑟 2 cos 45°
2 2 2 y
Φ′𝐵 = 0.360 𝑇 𝜋 × 0.250 𝑚
2
Φ′𝐵 = 0.0499 Wb
x
114
Example
A conducting circular loop of radius 0.250 m is placed in the xy-
plane in a uniform magnetic field of 0.360 T that points in the
positive z-direction, the same direction as the normal to the plane.
1. Calculate the magnetic flux through the loop.
2. Suppose the loop is rotated clockwise around the x-axis, so the
normal direction now points at a 45.0° angle with respect to the
z-axis. Recalculate the magnetic flux through the loop.
3. What is the change in flux due to the rotation of the loop?

Solution
3. ΔΦ𝐵 = Φ′𝐵 − Φ𝐵
ΔΦ𝐵 = 0.0706 Wb − 0.0499Wb
ΔΦ𝐵 = −0.0207 Wb
115
Magnetic Flux: Exercise
 A hexagon-shaped loop with area 0.5 m2 is
placed in a uniform B-field of 2 T such that
the loop’s normal is parallel to B.
a) Calculate ΦB through the hexagon.
b) Suppose the B-field strength is halved,.
Calculate ΦB and ΔΦB .
c) Suppose the loop is rotated 45° as shown
while B = 2T. Calculate ΦB and ΔΦB .

Solution

a) Φ𝐵1 𝐵𝐴 cos 𝜃 = 𝐵𝐴 cos 0° = (2𝑇)(0.5 𝑚2) = 1.0 𝑊𝑏


=
Φ𝐵1
b) Φ𝐵2 = 0.5 𝑊𝑏, ΔΦB = Φ𝐵2 – Φ𝐵1 = (0.5– 1.0)𝑊𝑏 = – 0.5 𝑊𝑏
= 2
c) ΦB2 = 𝐵𝐴 cos 𝜃 = 𝐵𝐴 cos 45° = (2𝑇)(0.5 𝑚2)(0.707) = 0.707 𝑊𝑏
ΔΦ𝐵 = Φ𝐵2 – Φ𝐵1 = (1.0 − 0.707)𝑊𝑏 = 0.293 𝑊𝑏
Faraday’s Law of Induction
• We’ve seen that electric currents can produce magnetic
fields.

• Can we use magnetic fields to generate electric currents?

• The answer is YES -- as discovered by Michael Faraday.

Magnetic fields can be used to generate


electric fields.

But, How?
Faraday’s Experiment
• Faraday tested this theory by winding two
solenoids around the same doughnut
shape of soft iron.
Faraday’s Experiment

He closed switch. Just after switch is on, he observed induced


current in secondary circuit as indicated by the ammeter.
Faraday’s Experiment

He noted that while the switch remained closed, the induced current
disappears as shown by the zero reading of the ammeter.
Faraday’s Experiment

He opened the switch. Immediately after, he saw an induced current


in the OPPOSITE direction as registered by the ammeter.
Faraday’s Experiment

He observed no induced current while the switch remained opened


as shown by the ammeter.
A second Experiment
 A loop of wire is connected to a
sensitive ammeter

 When the magnet is being


moved into the loop, current is
registered by the ammeter

 When the magnet is being


moved out of the loop, current
is registered by the ammeter in
the opposite direction.

 When the magnet is being held


still, no current is registered.
A third Experiment
 A loop of wire rotated by
some external means in a
magnetic field.

 The lighting bulb indicates


that current is induced in
the rotating loop

 If the loop is stationary the


bulb does not light up.
Faraday’s Law of induction
In all cases, the induced current is a result of a magnetic flux
that changes with time.
We know that the magnetic flux is defined as

Φ𝐵 = 𝐵𝐴 cos 𝜃

Thus the magnetic flux thru a coil changes if we change:


1. The magnetic field strength, B, that links the coil
2. The area of the coil
3. The orientation of the coil

Changing B Changing θ
Faraday’s Law of induction
The induced current must be
driven by an induced emf in
the coil.

Faraday’s Law of induction :

The emf induced in a loop is


directly proportional to the
time rate of change of the
magnetic flux through the
loop .
ΔΦ𝐵
ℰ=−
Δ𝑡

The sign of the emf is obtained from Lenz’s law


Lenz’s Law (direction of the induced emf)
 Lenz’s law:
“The induced current in a loop is in the direction that
creates a magnetic field that opposes the change in
magnetic flux through the area enclosed by the loop.

i.e. the induced current tends to keep the original


magnetic flux through the circuit from changing.
ΔΦ𝐵
𝜀=−
Δ𝑡

Lenz’s Law
Lenz’s Law: Example
 As the solenoid switches on,
creating upward magnetic
flux through the loop, the
I
current generated in the loop
will add downward flux.

 What if the solenoid is Solenoid just


switched off? switching on
Lenz’s Law: Example
 As the N pole moves up
towards the loop, the current
induced generates an N pole
underneath to repel and slow
down the approaching
magnet. I
N
Magnet
 What if the N pole recedes? moving up
The S pole approaches the
S
loop?
Lenz’s Law: Example
A rectangular loop is
moving through a uniform
magnetic field as shown. Is
there an induced current in
the rectangular loop?
Discuss

What is the direction of the


induced emf, if any?
Motional emf, the concept
⚫ A motional emf is the emf
induced in a conductor moving
through a magnetic field
⚫ The electrons in the conductor
experience a force, F = 𝑞𝑣𝐵 that
is directed along ℓ
⚫ Charges are accumulated at the
ends of the conductor to create
an electric field inside the
conductor to stop further
charge transportation.
⚫ In equilibrium
𝐹𝐵 = 𝐹𝐸 or 𝑞𝑣𝐵 = 𝑞𝐸
Motional emf, the calculation
 The equilibrium condition is then

𝐸 = 𝑣𝐵
 The emf, potential difference, is:

ℰ = Δ𝑉 = 𝐸𝑙 = 𝑣𝐵𝑙

 As long as the bar is kept moving


with a velocity 𝑣, the motional emf is
maintained to be
ℰ = 𝒗𝑩ℓ
Pulling a Loop out of a Field
 A square loop of side ℓ is moving
at speed v out of a region of B perpendicular to loop
uniform field B. speed v
 The induced emf is

ΔΦ𝐵 𝐵𝑣ℓΔ𝑡 ℓ
ℰ= = = 𝐵𝑣ℓ
Δ𝑡 Δ𝑡
 What about direction?
 The downward flux through
the loop is decreasing, the loop
In time Δt the loop will move
will try to oppose this by
distance v Δt, so the area of lost
making more downward flux magnetic flux will be v Δt ℓ.
(Lenz’s law).
Example: Motional emf powering a resistor
Condition:
A bar moving on two
rails. The bar and the
rails have negligible
resistance. A resistor of
R is connected to the I
end of the two rails.
Result:
The emf = vBℓ, so the
Equivalent circuit
current I = vBℓ /R diagram
Bar moved by 𝐅Ԧ𝑎𝑝𝑝
Two issues need attention:
1. The moving bar experiences a force from the field, FB=IℓB
2. The magnetic flux in the enclosed area is ΦB=xℓB, and it is
changing with time as
ΔΦ𝐵 Δ Δ𝑥
= 𝑥ℓ𝐵 = ℓ𝐵 = 𝑣𝐵ℓ = 𝑒𝑚𝑓
Δ𝑡 Δ𝑡 Δ𝑡
Example, what is the terminal velocity?
A bar of mass m sides on two vertical
rails. A resistor is connected to the end of
the rails. The bar is released at t = t0.
(a) calculate the velocity of the bar at
time t,
(b) what is the terminal velocity?
Assuming that the rails are long and the
magnetic field is large. I
m
(a) When the bar starts to fall, an emf is 𝐅Ԧ𝐺 = 𝑚𝑔𝑖Ƹ
induced:
𝑒𝑚𝑓 = 𝑣𝐵 ℓ
The induced current is: 𝐼 = 𝑣𝐵ℓ/𝑅
2
The magnetic force on the bar points 𝑣 ℓ𝐵
𝐅Ԧ𝐵 = −𝐼ℓ𝐵𝑖Ƹ = − 𝑖Ƹ
opposite to the gravitational force: 𝑅
Example, what is the terminal velocity?
 Construct the equation of motion:

𝑣 ℓ𝐵 2
Ԧ Ԧ
𝐅𝐺 + 𝐅𝐵 = 𝑚𝑔 − 𝑖Ƹ = 𝑚𝐚
𝑅
𝑣 ℓ𝐵 2 𝑑𝑣
𝑚𝑔 − =𝑚
𝑅 𝑑𝑡 I
m
𝐅Ԧ𝐺 = 𝑚𝑔𝑖Ƹ
 Solve this equation
𝑑𝑣 𝑑𝑡 𝑚𝑅
= − ,𝜏 ≡
𝑣 − 𝜏𝑔 𝜏 ℓ𝐵 2
𝑡
−𝜏
𝑣 = 𝜏𝑔 1 − 𝑒 ,∵ 𝑣 𝑡 = 0 = 0

 For (b), the terminal velocity is 𝜏𝑔 when 𝑡 → ∞


136
Example
 A coil with 25 turns of wire is wrapped on a
frame with a square cross section 1.80 cm on
a side. Each turn has the same area, equal to
that of the frame, and the total resistance of
the coil is 0.350 Ω. A uniform magnetic field
is applied perpendicular to the plane of the
coil.
a) While the field changes uniformly from 0.00 T to 0.500 T in
0.800 s, what is the induced emf in the coil?
b) Find the magnitude and direction of the induced current in the
coil while the field is changing.

137
Solution (a)
Area of loop: 𝐴 = 1.80 × 10−2 cm 2 = 3.24 × 10−4 m2

Flux at 𝑡 = 0:
Φ𝐵,𝑖 = 𝐵𝑖 𝐴 = 0.00 𝑊𝑏
Flux at 𝑡 = 0.800𝑠:
Φ𝐵,𝑓 = 𝐵𝑓 𝐴 = 0.500 T × 3.24 × 10−4 m2 = 1.62 × 10−4 Wb

Change in flux: ΔΦ𝐵 = 1.62 × 10−4 Wb

Induced emf:
ΔΦ𝐵 1.62 × 10−4 𝑊𝑏
ℰ = −𝑁 = −25 𝑡𝑢𝑟𝑛𝑠
Δ𝑡 0.800 𝑠
−𝟑
𝓔 = −𝟓. 𝟎𝟔 × 𝟏𝟎 𝑽

138
Solution (b)
 Induced current:

ℰ 5.06 × 10−3 𝑉
𝐼= = = 1.45 × 10−2 A
𝑅 0.350 Ω

 The current must be in a clockwise direction as viewed


from above the coil.
 This is because, by Lenz’s law, the flux is positive and
increasing. This is opposed by a negative flux from a
downward magnetic field that could exist only if the
induced current is clockwise as seen from above.

139

You might also like